Calculating Arc Length of y=x^2 from 0 to 10 using Trig Substitution"

Click For Summary
To calculate the arc length of the function y=x^2 from 0 to 10, the integral used is ∫ from 0 to 10 of √(1+(2x)²) dx. The discussion involves using trigonometric substitution, specifically setting 2x = tan(θ), which transforms the integrand. However, there is confusion regarding the integration of secant and the proper handling of dx after substitution. The correct approach requires remembering to include the derivative of the substitution, leading to dx = (1/2)sec²(θ)dθ. The thread highlights the importance of careful substitution and integration steps in solving arc length problems.
negatifzeo
Messages
66
Reaction score
0

Homework Statement


Find the arclength of the function y=x^2 when x is between 0 and 10.

Homework Equations


Arclength here is \int_{0}^{10} \sqrt{1+(2x)^2} dx
(It's intended to be the integral from 0 to 10 of the quare root of 1+(2x)^2. My latex skills suck.)

The Attempt at a Solution


Trig substitution. 2x= tan (theta)

Integrate secant theta
Which is ln(sec(theta)+tan(theta))

Substituting theta back in for arcsin(2x)
ln(sec(arctan(2x))+tan(arctan(2x))) evaluated from 10 to 0. Solving for arctan(2x) my final answer is
ln(sqrt(1+4x^2)+2x) evaluated from 0 to 10. I know this is wrong as it is much to short of an arc length but I don't know where I went wrong. Any clues?






The Attempt at a Solution

 
Last edited by a moderator:
Physics news on Phys.org
negatifzeo said:

Homework Statement


Find the arclength of the function y=x^2 when x is between 0 and 10.

Homework Equations


Arclength here is \int_{0}^{10} \sqrt{1+(2x)^2} dx
(It's intended to be the integral from 0 to 10 of the quare root of 1+(2x)^2. My latex skills suck.)

I've edited your post to fix your TeX.

The Attempt at a Solution


Trig substitution. 2x= tan (theta)

That's fine.

Integrate secant theta
Which is ln(sec(theta)+tan(theta))

That's not fine. Why would you integrate \sec (\theta )?
 
Tom Mattson said:
I've edited your post to fix your TeX.



That's fine.



That's not fine. Why would you integrate \sec (\theta )?

Well, when I use trig substitution here I change 2x to tan(theta). This changes the integrand to sqrt(1+tan^2(theta)). The trig identity says that 1 + tangent squared equals secant squared, and since it is the quare root of that it just becomes secant.
 
But you've forgotten about the dx.
 
Oh, duh. Wait, why am I having a hard time remembering how to get the dx here? Does dx here equal 1/2*sec^2(theta) d(theta)?
 
Yes, that's right.
 
Question: A clock's minute hand has length 4 and its hour hand has length 3. What is the distance between the tips at the moment when it is increasing most rapidly?(Putnam Exam Question) Answer: Making assumption that both the hands moves at constant angular velocities, the answer is ## \sqrt{7} .## But don't you think this assumption is somewhat doubtful and wrong?

Similar threads

  • · Replies 2 ·
Replies
2
Views
2K
Replies
2
Views
2K
Replies
7
Views
2K
  • · Replies 5 ·
Replies
5
Views
2K
  • · Replies 6 ·
Replies
6
Views
2K
Replies
2
Views
1K
  • · Replies 1 ·
Replies
1
Views
1K
  • · Replies 1 ·
Replies
1
Views
2K
Replies
3
Views
2K
  • · Replies 6 ·
Replies
6
Views
2K